Những câu hỏi liên quan
Trần Lâm Thiên Hương
Xem chi tiết
Trần Lâm Thiên Hương
15 tháng 5 2018 lúc 21:03

Mình nhầm, phải là \(\le\frac{1}{3}\)mọi người làm giúp mình với mình cần gấp

Bình luận (0)
zZz Cool Kid_new zZz
1 tháng 8 2020 lúc 19:31

Theo BĐT Cauchy Schwarz và các biến đổi cơ bản ta dễ có được:
\(\frac{a^2}{\left(2a+b\right)\left(2a+c\right)}=\frac{a^2}{2a\left(a+b+c\right)+2a^2+bc}=\frac{1}{9}\left[\frac{\left(2a+a\right)^2}{2a\left(a+b+c\right)+2a^2+bc}\right]\)

\(\le\frac{1}{9}\left[\frac{4a^2}{2a\left(a+b+c\right)}+\frac{a^2}{2a^2+bc}\right]=\frac{1}{9}\left(\frac{2a}{a+b+c}+\frac{a^2}{2a^2+bc}\right)\)

\(\Rightarrow LHS\le\frac{1}{9}\left(2+\frac{a^2}{2a^2+bc}+\frac{b^2}{2b^2+ca}+\frac{c^2}{2c^2+ab}\right)\)

Tiếp tục theo BĐT Cauchy Schwarz dạng Engel:

\(\frac{a^2}{a^2+2bc}+\frac{b^2}{b^2+2ca}+\frac{c^2}{c^2+2ab}\ge\frac{\left(a+b+c\right)^2}{\left(a+b+c\right)^2}=1\)

Ta thực hiện phép đổi biến thì:

\(\frac{ab}{ab+2c^2}+\frac{bc}{bc+2a^2}+\frac{ca}{ca+2b^2}\ge1\)

Đến đây là phần của bạn

Bình luận (0)
 Khách vãng lai đã xóa
tth_new
3 tháng 8 2020 lúc 19:10

(Vào thống kê hỏi đáp xem ảnh nhé! 2 cách, cách đầu dùng kỹ thuật uvw, cách kia là SOS)

Bình luận (0)
 Khách vãng lai đã xóa
Phạm Văn Việt
Xem chi tiết
Neet
Xem chi tiết
Akai Haruma
2 tháng 3 2017 lúc 0:34

Bài 3)

BĐT cần chứng minh tương đương với:

\(\left ( \frac{a}{a+b} \right )^2+\left ( \frac{b}{b+c} \right )^2+\left ( \frac{c}{c+a} \right )^2\geq \frac{1}{2}\left ( 3-\frac{a}{a+b}-\frac{b}{b+c}-\frac{c}{c+a} \right )\)

Để cho gọn, đặt \((x,y,z)=\left (\frac{b}{a},\frac{c}{b},\frac{a}{c}\right)\) \(\Rightarrow xyz=1\).

BĐT được viết lại như sau:

\(A=2\left [ \frac{1}{(x+1)^2}+\frac{1}{(y+1)^2}+\frac{1}{(z+1)^2} \right ]+\frac{1}{x+1}+\frac{1}{y+1}+\frac{1}{z+1}\geq 3\) \((\star)\)

Ta nhớ đến hai bổ đề khá quen thuộc sau:

Bổ đề 1: Với \(a,b>0\) thì \(\frac{1}{(a+1)^2}+\frac{1}{(b+1)^2}\geq \frac{1}{ab+1}\)

Cách CM rất đơn giản, Cauchy - Schwarz:

\((a+1)^2\leq (a+b)(a+\frac{1}{b})\Rightarrow \frac{1}{(a+1)^2}\geq \frac{b}{(a+b)(ab+1)}\)

Tương tự với biểu thức còn lại và cộng vào thu được đpcm

Bổ đề 2: Với \(x,y>0,xy\geq 1\) thì \(\frac{1}{x^2+1}+\frac{1}{y^2+1}\geq \frac{2}{xy+1}\)

Cách CM: Quy đồng ta có đpcm.

Do tính hoán vị nên không mất tổng quát giả sử \(z=\min (x,y,z)\)

\(\Rightarrow xy\geq 1\). Áp dụng hai bổ đề trên:

\(A\geq 2\left [ \frac{1}{xy+1}+\frac{1}{(z+1)^2} \right ]+\frac{2}{\sqrt{xy}+1}+\frac{1}{z+1}=2\left [ \frac{z}{z+1}+\frac{1}{(z+1)^2} \right ]+\frac{2\sqrt{z}}{\sqrt{z}+1}+\frac{1}{z+1}\)

\(\Leftrightarrow A\geq \frac{2(z^2+z+1)}{(z+1)^2}+\frac{1}{z+1}+2-\frac{2}{\sqrt{z}+1}\geq 3\)

\(\Leftrightarrow 2\left [ \frac{z^2+z+1}{(z+1)^2}-\frac{3}{4} \right ]+\frac{1}{z+1}-\frac{1}{2}-\left ( \frac{2}{\sqrt{z}+1}-1 \right )\geq 0\)

\(\Leftrightarrow \frac{(z-1)^2}{2(z+1)^2}-\frac{z-1}{2(z+1)}+\frac{z-1}{(\sqrt{z}+1)^2}\geq 0\Leftrightarrow (z-1)\left [ \frac{1}{(\sqrt{z}+1)^2}-\frac{1}{(z+1)^2} \right ]\geq 0\)

\(\Leftrightarrow \frac{\sqrt{z}(\sqrt{z}-1)^2(\sqrt{z}+1)(z+\sqrt{z}+2)}{(\sqrt{z}+1)^2(z+1)^2}\geq 0\) ( luôn đúng với mọi \(z>0\) )

Do đó \((\star)\) được cm. Bài toán hoàn tất.

Dấu bằng xảy ra khi \(a=b=c\)

P/s: Nghỉ tuyển lâu rồi giờ mới gặp mấy bài BĐT phải động não. Khuya rồi nên xin phép làm bài 3 trước. Hai bài kia xin khiếu. Nếu làm đc chắc tối mai sẽ post.

Bình luận (1)
Lightning Farron
2 tháng 3 2017 lúc 18:11

Bài 1:

Cho \(a=b=c=\dfrac{1}{\sqrt{3}}\). Khi đó \(M=\sqrt{3}-2\)

Ta sẽ chứng minh nó là giá trị nhỏ nhất

Thật vậy, đặt c là giá trị nhỏ nhất của a,b,c. Khi đó, ta cần chứng minh

\(\frac{a^2}{b}+\frac{b^2}{c}+\frac{c^2}{a}-\frac{2(a^2+b^2+c^2)}{\sqrt{ab+ac+bc}}\geq(\sqrt3-2)\sqrt{ab+ac+bc}\)

\(\Leftrightarrow\sqrt{ab+ac+bc}\left(\frac{a^2}{b}+\frac{b^2}{c}+\frac{c^2}{a}-\sqrt{3(ab+ac+bc)}\right)\geq2(a^2+b^2+c^2-ab-ac-bc)\)

\(\Leftrightarrow\frac{a^2}{b}+\frac{b^2}{a}-a-b+\frac{b^2}{c}+\frac{c^2}{a}-\frac{b^2}{a}-c+a+b+c-\sqrt{3(ab+ac+bc)}\geq\)

\(\geq2((a-b)^2+(c-a)(c-b))\)

\(\Leftrightarrow(a-b)^2\left(\frac{1}{a}+\frac{1}{b}-2\right)+(c-a)(c-b)\left(\frac{1}{a}+\frac{b}{ac}-2\right)+a+b+c-\sqrt{3(ab+ac+bc)}\geq0\)

Đúng bởi \(\frac{1}{a}+\frac{1}{b}-2>0;\frac{1}{a}+\frac{b}{ac}-2\geq\frac{1}{a}+\frac{1}{a}-2>0\)

\(a+b+c-\sqrt{3(ab+ac+bc)}=\frac{(a-b)^2+(c-a)(c-b)}{a+b+c+\sqrt{3(ab+ac+bc)}}\geq0\)

BĐT đã được c/m. Vậy \(M_{Min}=\sqrt{3}-2\Leftrightarrow a=b=c=\dfrac{1}{\sqrt{3}}\)

P/s: Nhìn qua thấy ngon mà làm mới thấy thật sự là "choáng"

Bình luận (3)
Hung nguyen
2 tháng 3 2017 lúc 10:57

Câu 1/ Ta có

\(ab+bc+ca\le\frac{\left(a+b+c\right)^2}{3}\)

\(\Leftrightarrow1\le\frac{\left(a+b+c\right)^2}{3}\)

\(\Leftrightarrow\left(a+b+c\right)^2\ge3\)

\(\Leftrightarrow\sqrt{3}\le a+b+c< 3\)

Ta có: \(M=\frac{a^2\left(1-2b\right)}{b}+\frac{b^2\left(1-2c\right)}{c}+\frac{c^2\left(1-2a\right)}{a}\)

\(=\frac{a^2}{b}+\frac{b^2}{c}+\frac{c^2}{a}-2\left(a^2+b^2+c^2\right)\)

\(\ge\frac{\left(a+b+c\right)^2}{a+b+c}-2\left(a^2+b^2+c^2\right)-4\left(ab+bc+ca\right)+4\left(ab+bc+ca\right)\)

\(=a+b+c-2\left(a+b+c\right)^2+4\) (1)

Đặt \(a+b+c=x\left(\sqrt{3}\le x< 3\right)\)

Ta tìm GTNN của hàm số: \(y=-2x^2+x+4\)

\(\Rightarrow y'=-4x+1=0\)

\(\Rightarrow x=\frac{1}{4}=0,25\)

Thế x lần lược các giá trị \(\left\{\begin{matrix}x=0,25\\x=\sqrt{3}\end{matrix}\right.\)

\(\Rightarrow\left\{\begin{matrix}y=4,125\\y=-2+\sqrt{3}\end{matrix}\right.\)

\(\Rightarrow y_{min}=-2+\sqrt{3}\) đạt cực trị tại \(x=\sqrt{3}\) (2)

Từ (1) và (2) ta suy ra GTNN của M là \(-2+\sqrt{3}\) tại \(a=b=c=\frac{1}{\sqrt{3}}\)

Bình luận (7)
Nguyễn Thị Minh Nguyệt
Xem chi tiết
tth_new
6 tháng 9 2020 lúc 16:34

Bài này không đúng nhé. Với a = b = c = 1 thì bất đẳng thức sai. Tuy nhiên bài này đúng theo chiều ngược lại.

Bình luận (0)
 Khách vãng lai đã xóa
Phan Nghĩa
7 tháng 9 2020 lúc 20:18

Ta sẽ chứng minh bất đẳng thức phụ sau đây \(x^2+y^2+z^2\ge xy+yz+zx\)

\(< =>2\left(x^2+y^2+z^2\right)\ge2\left(xy+yz+zx\right)\)

\(< =>2x^2+2y^2+2z^2-2xy-2yz-2zx\ge0\)

\(< =>\left(x-y\right)^2+\left(y-z\right)^2+\left(z-x\right)^2\ge0\)*đúng*

Đặt \(\left\{2a+2b-c;2b+2c-a;2c+2a-b\right\}\rightarrow\left\{x;y;z\right\}\)

Vì a,b,c là ba cạnh của 1 tam giác nên x,y,z dương 

Ta có : \(x^2+y^2+z^2=9\left(a^2+b^2+c^2\right)\)

\(x+y=c+a+4b\)\(y+z=a+b+4c\)\(z+x=b+c+4a\)

Bất đẳng thức cần chứng minh quy về : \(\frac{x^3}{y+z}+\frac{y^3}{x+z}+\frac{z^3}{x+y}\ge\frac{x^2+y^2+z^2}{2}\)

Áp dụng bất đẳng thức AM-GM ta có : 

\(\frac{x^3}{y+z}+\frac{x\left(y+z\right)}{4}\ge2\sqrt{\frac{x^3.x\left(y+z\right)}{\left(y+z\right)4}}=2\sqrt{\frac{x^4}{4}}=2\frac{x^2}{2}=x^2\)

\(\frac{y^3}{x+z}+\frac{y\left(x+z\right)}{4}\ge2\sqrt{\frac{y^3.y\left(x+z\right)}{\left(x+z\right)4}}=2\sqrt{\frac{y^4}{4}}=2\frac{y^2}{2}=y^2\)

\(\frac{z^3}{x+y}+\frac{z\left(x+y\right)}{4}\ge2\sqrt{\frac{z^3.z\left(x+y\right)}{\left(x+y\right)4}}=2\sqrt{\frac{z^4}{4}}=2\frac{z^2}{2}=z^2\)

Cộng theo vế các bất đẳng thức cùng chiều ta được :

\(\frac{x^3}{y+z}+\frac{y^3}{x+z}+\frac{z^3}{x+y}+\frac{x\left(y+z\right)}{4}+\frac{y\left(x+z\right)}{4}+\frac{z\left(x+y\right)}{4}\ge x^2+y^2+z^2\)

\(< =>\frac{x^3}{y+z}+\frac{y^3}{x+z}+\frac{z^3}{x+y}+\frac{xy+yz+zx+xy+yz+zx}{4}\ge x^2+y^2+z^2\)

\(< =>\frac{x^3}{y+z}+\frac{y^3}{x+z}+\frac{z^3}{x+y}+\frac{xy+yz+zx}{2}\ge x^2+y^2+z^2\)

\(< =>\frac{x^3}{y+z}+\frac{y^3}{x+z}+\frac{z^3}{x+y}\ge x^2+y^2+z^2-\frac{xy+yz+zx}{2}\)

Sử dụng bất đẳng thức phụ \(x^2+y^2+z^2\ge xy+yz+zx\)khi đó ta được :

\(\frac{x^3}{y+z}+\frac{y^3}{x+z}+\frac{z^3}{y+x}\ge x^2+y^2+z^2-\frac{x^2+y^2+z^2}{2}\)

\(< =>\frac{x^3}{y+z}+\frac{y^3}{z+x}+\frac{z^3}{x+y}\ge\frac{x^2+y^2+z^2}{2}\left(đpcm\right)\)

Đẳng thức xảy ra khi và chỉ khi \(x=y=z< =>a=b=c\)

Vậy ta có điều phải chứng minh

Bình luận (0)
 Khách vãng lai đã xóa
Đức Hiếu Nguyễn
Xem chi tiết
Kiệt Nguyễn
9 tháng 6 2020 lúc 13:58

Không mất tính tổng quát, chuẩn hóa a + b + c = 1

Khi đó, ta cần chứng minh: \(\frac{\left(a+1\right)^2}{2a^2+\left(1-a\right)^2}+\frac{\left(b+1\right)^2}{2b^2+\left(1-b\right)^2}+\frac{\left(c+1\right)^2}{2c^2+\left(1-c\right)^2}\le8\)

Xét bất đẳng thức phụ: \(\frac{\left(x+1\right)^2}{2x^2+\left(1-x\right)^2}\le4x+\frac{4}{3}\)(*)

Thật vậy: (*)\(\Leftrightarrow\frac{\left(3x-1\right)^2\left(4x+1\right)}{2x^2+\left(1-x\right)^2}\ge0\)*đúng*

Áp dụng, ta được: \(\frac{\left(a+1\right)^2}{2a^2+\left(1-a\right)^2}+\frac{\left(b+1\right)^2}{2b^2+\left(1-b\right)^2}+\frac{\left(c+1\right)^2}{2c^2+\left(1-c\right)^2}\)\(\le4\left(a+b+c\right)+4=4.1+4=8\)

Vậy bất đẳng thức được chứng minh

Đẳng thức xảy ra khi a = b = c

Bình luận (0)
 Khách vãng lai đã xóa
Trần Phúc Khang
25 tháng 7 2019 lúc 15:44

Chuẩn hóa ta có : \(a+b+c=3\)

=> \(\frac{\left(2a+b+c\right)^2}{2a^2+\left(b+c\right)^2}=\frac{\left(a+3\right)^2}{2a^2+\left(3-a\right)^2}=\frac{a^2+6a+9}{3\left(a^2-2a+3\right)}\)

Xét\(\frac{a^2+6a+9}{3\left(a^2-2a+3\right)}\le\frac{4}{3}a+\frac{4}{3}\)

<=> \(a^2+6a+9\le4\left(a+1\right)\left(a^2-2a+3\right)\)

<=> \(4a^3-5a^2-2a+3\ge0\)

<=> \(\left(a-1\right)^2\left(4a+3\right)\ge0\)luôn đúng

Khi đó 

\(VT\le\frac{4}{3}\left(a+b+c\right)+4=\frac{4}{3}.3+4=8\)(ĐPCM)

Dấu bằng xảy ra khi a=b=c

Bình luận (0)
ミ★Zero ❄ ( Hoàng Nhật )
2 tháng 5 2020 lúc 19:50

bài lớp 10 em chưa hok nha anh

Bình luận (0)
 Khách vãng lai đã xóa
Trần Anh Thơ
Xem chi tiết
Nguyễn Việt Lâm
6 tháng 6 2020 lúc 13:14

\(P=\frac{a^2}{b^2+2bc}+\frac{b^2}{c^2+2ac}+\frac{c^2}{a^2+2ab}\ge\frac{\left(a+b+c\right)^2}{a^2+b^2+c^2+2ab+2bc+2ca}=\frac{\left(a+b+c\right)^2}{\left(a+b+c\right)^2}=1\)

Dấu "=" xảy ra khi \(a=b=c\)

Bình luận (0)
Lê Minh Đức
Xem chi tiết
Akai Haruma
9 tháng 10 2017 lúc 17:28

Ta có \(a^2b^2+b^2c^2+c^2a^2\geq a^2b^2c^2\Leftrightarrow \frac{1}{a^2}+\frac{1}{b^2}+\frac{1}{c^2}\geq 1\)

BĐT cần chứng minh tương đương với \(\frac{\frac{1}{c^3}}{\frac{1}{a^2}+\frac{1}{b^2}}+\frac{\frac{1}{b^3}}{\frac{1}{a^2}+\frac{1}{c^2}}+\frac{\frac{1}{a^3}}{\frac{1}{b^2}+\frac{1}{c^2}}\geq \frac{\sqrt{3}}{2}\)

Đặt \((\frac{1}{a},\frac{1}{b},\frac{1}{c})=(x,y,z)\). Bài toán trở thành: 

Cho \(x,y,z>0|x^2+y^2+z^2\geq 1\). CMR \(P=\frac{x^3}{y^2+z^2}+\frac{y^3}{z^2+x^2}+\frac{z^3}{x^2+y^2}\geq \frac{\sqrt{3}}{2}\)

Lời giải:

 Áp dụng BĐT Cauchy -Schwarz:

\(P=\frac{x^4}{xy^2+xz^2}+\frac{y^4}{yz^2+yx^2}+\frac{z^4}{zx^2+zy^2}\geq \frac{(x^2+y^2+^2)^2}{x^2(y+z)+y^2(x+z)+z^2(x+y)}\) (1)

Không mất tính tổng quát, giả sử \(x\geq y\geq z\Rightarrow x^2\geq y^2\geq z^2\) 

Và \(y+z\leq z+x\leq x+y\). Khi đó, áp dụng BĐT Chebyshev: 

\(3[x^2(y+z)+y^2(x+z)+z^2(x+y)]\leq (x^2+y^2+z^2)(y+z+x+z+x+y)\)

\(\Leftrightarrow x^2(y+z)+y^2(x+z)+z^2(x+y)\leq \frac{2(x^2+y^2+z^2)(x+y+z)}{3}\)

Theo hệ quả của BĐT Am-Gm thì: \((x+y+z)^2\leq 3(x^2+y^2+z^2)\Rightarrow x+y+z\leq \sqrt{3(x^2+y^2+z^2)}\)

\(\Rightarrow x^2(y+z)+y^2(x+z)+z^2(x+y)\leq \frac{2(x^2+y^2+z^2)\sqrt{3(x^2+y^2+z^2)}}{3}\) (2)

Từ (1),(2) suy ra \(P\geq \frac{3(x^2+y^2+z^2)^2}{2(x^2+y^2+z^2)\sqrt{3(x^2+y^2+z^2)}}=\frac{\sqrt{3(x^2+y^2+z^2)}}{2}\geq \frac{\sqrt{3}}{2}\)

Ta có đpcm

Dáu bằng xảy ra khi \(x=y=z=\frac{1}{\sqrt{3}}\Leftrightarrow a=b=c=\sqrt{3}\)

Bình luận (0)
Kiệt Nguyễn
5 tháng 5 2020 lúc 12:58

Đặt \(x=\frac{1}{a};y=\frac{1}{b};z=\frac{1}{c}\)

Khi đó giả thiết được viết lại là \(x^2+y^2+z^2\ge1\)và ta cần chứng minh \(\frac{x^3}{y^2+z^2}+\frac{y^3}{z^2+x^2}+\frac{z^3}{x^2+y^2}\ge\frac{\sqrt{3}}{2}\)(*)

Áp dụng BĐT Bunhiacopxki dạng phân thức, ta được:

\(VT_{\left(^∗\right)}=\frac{x^4}{x\left(y^2+z^2\right)}+\frac{y^4}{y\left(z^2+x^2\right)}+\frac{z^4}{z\left(x^2+y^2\right)}\)\(\ge\frac{\left(x^2+y^2+z^2\right)^2}{x\left(y^2+z^2\right)+y\left(z^2+x^2\right)+z\left(x^2+y^2\right)}\)

Đến đây ta đi chứng minh \(\frac{\left(x^2+y^2+z^2\right)^2}{x\left(y^2+z^2\right)+y\left(z^2+x^2\right)+z\left(x^2+y^2\right)}\ge\frac{\sqrt{3}}{2}\)

\(\Leftrightarrow2\left(x^2+y^2+z^2\right)^2\)\(\ge\sqrt{3}\left[x\left(y^2+z^2\right)+y\left(z^2+x^2\right)+z\left(x^2+y^2\right)\right]\)

Ta có: \(x\left(y^2+z^2\right)=\frac{1}{\sqrt{2}}\sqrt{2x^2\left(y^2+z^2\right)\left(y^2+z^2\right)}\)\(\le\frac{1}{\sqrt{2}}\sqrt{\left(\frac{2x^2+y^2+z^2+y^2+z^2}{3}\right)^3}\)

\(=\frac{2\sqrt{3}}{9}\left(x^2+y^2+z^2\right)\sqrt{x^2+y^2+z^2}\)

Tương tự ta có: \(y\left(z^2+x^2\right)\le\frac{2\sqrt{3}}{9}\left(x^2+y^2+z^2\right)\sqrt{x^2+y^2+z^2}\)

\(z\left(x^2+y^2\right)\le\frac{2\sqrt{3}}{9}\left(x^2+y^2+z^2\right)\sqrt{x^2+y^2+z^2}\)

Cộng theo vế của 3 BĐT trên, ta được: 

\(\text{∑}_{cyc}\left[x\left(y^2+z^2\right)\right]\le\frac{2\sqrt{3}}{3}\left(x^2+y^2+z^2\right)\sqrt{x^2+y^2+z^2}\)

\(\Leftrightarrow\sqrt{3}\text{∑}_{cyc}\left[x\left(y^2+z^2\right)\right]\le2\left(x^2+y^2+z^2\right)\sqrt{x^2+y^2+z^2}\)

Cuối cùng ta cần chứng minh được

\(2\left(x^2+y^2+z^2\right)\sqrt{x^2+y^2+z^2}\le2\left(x^2+y^2+z^2\right)^2\)

\(\Leftrightarrow x^2+y^2+z^2\ge1\)(đúng)

Đẳng thức xảy ra khi \(x=y=z=\frac{1}{\sqrt{3}}\Rightarrow a=b=c=\sqrt{3}\)

Bình luận (0)
 Khách vãng lai đã xóa
Nguyễn Lâm Ngọc
Xem chi tiết
lord huy
28 tháng 9 2017 lúc 20:46

moi nguoi oi hom truoc minh hoc tap hop cac so TN do thi co cua minh day nhu sau 

vd: A={xeN/3<x<9}

thi minh liet ke ra la A=4,5,6,7,8 nhung sua bai lai ko dung 

co sua nhu vay A=3,4,5,6,7,8

ko biet hay sai mong ae giup minh

Bình luận (0)
Vũ Đoàn
30 tháng 9 2017 lúc 13:44

Áp dụng BĐT Cô-si \(ab\le\frac{\left(a+b\right)}{4}^2\)

=> \(\left(2a+b\right)\left(2c+b\right)\le\frac{4\left(a+b+c\right)^2}{4}=\left(a+b+c\right)^2\)

=> \(\frac{1}{\left(2a+b\right)\left(2c+b\right)}\ge\frac{1}{\left(a+b+c\right)^2}\)

Mấy cái kia làm tương tự cậu nhé 

Dấu "=" xảy ra khi và chỉ khi a=b=c=1

Bình luận (0)
bach nhac lam
Xem chi tiết
tthnew
11 tháng 11 2019 lúc 20:40

2/ Không mất tính tổng quát, giả sử \(c=min\left\{a,b,c\right\}\).

Nếu abc = 0 thì có ít nhất một số bằng 0. Giả sử c = 0. BĐT quy về: \(a^2+b^2\ge2ab\Leftrightarrow\left(a-b\right)^2\ge0\) (luôn đúng)

Đẳng thức xảy ra khi a = b; c = 0.

Nếu \(abc\ne0\). Chia hai vế của BĐT cho \(\sqrt[3]{\left(abc\right)^2}\)

BĐT quy về: \(\Sigma_{cyc}\sqrt[3]{\frac{a^4}{b^2c^2}}+3\ge2\Sigma_{cyc}\sqrt[3]{\frac{ab}{c^2}}\)

Đặt \(\sqrt[3]{\frac{a^2}{bc}}=x;\sqrt[3]{\frac{b^2}{ca}}=y;\sqrt[3]{\frac{c^2}{ab}}=z\Rightarrow xyz=1\)

Cần chúng minh: \(x^2+y^2+z^2+3\ge2\left(\frac{1}{x}+\frac{1}{y}+\frac{1}{z}\right)\)

\(\Leftrightarrow x^2+y^2+z^2+2xyz+1\ge2\left(xy+yz+zx\right)\) (1)

Theo nguyên lí Dirichlet thì trong 3 số x - 1, y - 1, z - 1 tồn tại ít nhất 2 số có tích không âm. Không mất tính tổng quát, giả sử \(\left(x-1\right)\left(y-1\right)\ge0\)

\(\Rightarrow2xyz\ge2xz+2yz-2z\). Thay vào (1):

\(VT\ge x^2+y^2+z^2+2xz+2yz-2z+1\)

\(=\left(x-y\right)^2+\left(z-1\right)^2+2xy+2xz+2yz\)

\(\ge2\left(xy+yz+zx\right)\)

Vậy (1) đúng. BĐT đã được chứng minh.

Đẳng thức xảy ra khi a = b = c hoặc a = b, c = 0 và các hoán vị.

Check giúp em vs @Nguyễn Việt Lâm, bài dài quá:(

Bình luận (0)
 Khách vãng lai đã xóa
tthnew
6 tháng 7 2020 lúc 7:23

Cách khác câu 2:Đặt \(\left(a,b,c\right)=\left(a^3,b^3,c^3\right)\)

Có: \(VT-VP=\frac{1}{6} \sum\, \left( 3\,{a}^{2}+4\,ab+2\,ac+3\,{b}^{2}+2\,bc \right) \left( a -b \right) ^{2} \left( a+b-c \right) ^{2}+\frac{2}{3} \sum \,{a}^{2}{b}^{2} \left( a -b \right) ^{2} \geq 0\)

Bất đẳng thức trên vẫn đúng trong trường hợp $a,b,c$ là các số thực.

Thật vậy ta chỉ cần chứng minh$:$

\(\frac{1}{6}\sum \left( 3\,{a}^{2}+4\,ab+2\,ac+3\,{b}^{2}+2\,bc \right) \left( a -b \right) ^{2} \left( a+b-c \right) ^{2} \geq 0\)

Chú ý \(\sum\left(a-b\right)\left(a+b-c\right)=0\)

Ta đưa về chứng minh: \(\sum (3\,{a}^{2}+4\,ab+2\,ac+3\,{b}^{2}+2\,bc) \geq 0 \,\,\,\,\,\,(1)\)

\(\sum \left( 3\,{a}^{2}+2\,ab+4\,ac+2\,bc+3\,{c}^{2} \right) \left( 3\,{a} ^{2}+4\,ab+2\,ac+3\,{b}^{2}+2\,bc \right) \geq 0 \,\,\,\,(2)\)

$(1)$ dễ chứng minh bằng tam thức bậc $2$.

Chứng minh $(2):$

$$\text{VT} = {\frac {196\, \left( a+b+c \right) ^{4}}{27}} + \sum{\frac { \left( a-b \right) ^{2} \left( 47\,a+26\,c+47\,b \right) ^{2}
}{2538}}+\sum {\frac {328\,{c}^{2} \left( a-b \right) ^{2}}{141}} \geq 0$$

Xong.

Bình luận (0)